05.12.2012 Views

Download Print PDF

Download Print PDF

Download Print PDF

SHOW MORE
SHOW LESS

You also want an ePaper? Increase the reach of your titles

YUMPU automatically turns print PDFs into web optimized ePapers that Google loves.

Volume 8 (2007), Issue 1, Article 15, 21 pp.<br />

THE EQUAL VARIABLE METHOD<br />

VASILE CÎRTOAJE<br />

DEPARTMENT OF AUTOMATION AND COMPUTERS<br />

UNIVERSITY OF PLOIE ¸STI<br />

BUCURE ¸STI 39, ROMANIA<br />

vcirtoaje@upg-ploiesti.ro<br />

Received 01 March, 2006; accepted 17 April, 2006<br />

Communicated by P.S. Bullen<br />

ABSTRACT. The Equal Variable Method (called also n−1 Equal Variable Method on the Mathlinks<br />

Site - Inequalities Forum) can be used to prove some difficult symmetric inequalities involving<br />

either three power means or, more general, two power means and an expression of form<br />

f(x1) + f(x2) + · · · + f(xn).<br />

Key words and phrases: Symmetric inequalities, Power means, EV-Theorem.<br />

2000 Mathematics Subject Classification. 26D10, 26D20.<br />

1. STATEMENT OF RESULTS<br />

In order to state and prove the Equal Variable Theorem (EV-Theorem) we require the following<br />

lemma and proposition.<br />

Lemma 1.1. Let a, b, c be fixed non-negative real numbers, not all equal and at most one of<br />

them equal to zero, and let x ≤ y ≤ z be non-negative real numbers such that<br />

x + y + z = a + b + c, x p + y p + z p = a p + b p + c p ,<br />

where p ∈ (−∞, 0] ∪ (1, ∞). For p = 0, the second equation is xyz = abc > 0. Then, there<br />

exist two non-negative real numbers x1 and x2 with x1 < x2 such that x ∈ [x1, x2]. Moreover,<br />

(1) if x = x1 and p ≤ 0, then 0 < x < y = z;<br />

(2) if x = x1 and p > 1, then either 0 = x < y ≤ z or 0 < x < y = z;<br />

(3) if x ∈ (x1, x2), then x < y < z;<br />

(4) if x = x2, then x = y < z.<br />

Proposition 1.2. Let a, b, c be fixed non-negative real numbers, not all equal and at most one<br />

of them equal to zero, and let 0 ≤ x ≤ y ≤ z such that<br />

059-06<br />

x + y + z = a + b + c, x p + y p + z p = a p + b p + c p ,


2 VASILE CÎRTOAJE<br />

where p ∈ (−∞, 0] ∪ (1, ∞). For p = 0, the second equation is xyz = abc > 0. Let f(u) be a<br />

differentiable function on (0, ∞), such that g(x) = f ′<br />

�<br />

x 1 �<br />

p−1 is strictly convex on (0, ∞), and<br />

let<br />

F3(x, y, z) = f(x) + f(y) + f(z).<br />

(1) If p ≤ 0, then F3 is maximal only for 0 < x = y < z, and is minimal only for<br />

0 < x < y = z;<br />

(2) If p > 1 and either f(u) is continuous at u = 0 or lim f(u) = −∞, then F3 is maximal<br />

u→0<br />

only for 0 < x = y < z, and is minimal only for either x = 0 or 0 < x < y = z.<br />

Theorem 1.3 (Equal Variable Theorem (EV-Theorem)). Let a1, a2, . . . , an (n ≥ 3) be fixed<br />

non-negative real numbers, and let 0 ≤ x1 ≤ x2 ≤ · · · ≤ xn such that<br />

x1 + x2 + · · · + xn = a1 + a2 + · · · + an,<br />

x p<br />

1 + x p<br />

2 + · · · + x p n = a p<br />

1 + a p<br />

2 + · · · + a p n,<br />

where p is a real number, p �= 1. For p = 0, the second equation is x1x2 · · · xn = a1a2 · · · an ><br />

0. Let f(u) be a differentiable function on (0, ∞) such that<br />

g(x) = f ′<br />

�<br />

x 1 �<br />

p−1<br />

is strictly convex on (0, ∞), and let<br />

Fn(x1, x2, . . . , xn) = f(x1) + f(x2) + · · · + f(xn).<br />

(1) If p ≤ 0, then Fn is maximal for 0 < x1 = x2 = · · · = xn−1 ≤ xn, and is minimal for<br />

0 < x1 ≤ x2 = x3 = · · · = xn;<br />

(2) If p > 0 and either f(u) is continuous at u = 0 or lim f(u) = −∞, then Fn is<br />

u→0<br />

maximal for 0 ≤ x1 = x2 = · · · = xn−1 ≤ xn, and is minimal for either x1 = 0 or<br />

0 < x1 ≤ x2 = x3 = · · · = xn.<br />

Remark 1.4. Let 0 < α < β. If the function f is differentiable on (α, β) and the function<br />

g(x) = f ′<br />

�<br />

x 1 �<br />

p−1<br />

holds true for x1, x2, . . . , xn ∈ (α, β).<br />

is strictly convex on (α p−1 , β p−1 ) or (β p−1 , α p−1 ), then the EV-Theorem<br />

By Theorem 1.3, we easily obtain some particular results, which are very useful in applications.<br />

Corollary 1.5. Let a1, a2, . . . , an (n ≥ 3) be fixed non-negative numbers, and let 0 ≤ x1 ≤<br />

x2 ≤ · · · ≤ xn such that<br />

x1 + x2 + · · · + xn = a1 + a2 + · · · + an,<br />

x 2 1 + x 2 2 + · · · + x 2 n = a 2 1 + a 2 2 + · · · + a 2 n.<br />

Let f be a differentiable function on (0, ∞) such that g(x) = f ′ (x) is strictly convex on (0, ∞).<br />

Moreover, either f(x) is continuous at x = 0 or lim<br />

x→0 f(x) = −∞. Then,<br />

Fn = f(x1) + f(x2) + · · · + f(xn)<br />

is maximal for 0 ≤ x1 = x2 = · · · = xn−1 ≤ xn, and is minimal for either x1 = 0 or<br />

0 < x1 ≤ x2 = x3 = · · · = xn.<br />

J. Inequal. Pure and Appl. Math., 8(1) (2007), Art. 15, 21 pp. http://jipam.vu.edu.au/


EQUAL VARIABLE METHOD 3<br />

Corollary 1.6. Let a1, a2, . . . , an (n ≥ 3) be fixed positive numbers, and let 0 < x1 ≤ x2 ≤<br />

· · · ≤ xn such that<br />

x1 + x2 + · · · + xn = a1 + a2 + · · · + an,<br />

1<br />

x1<br />

+ 1<br />

x2<br />

+ · · · + 1<br />

xn<br />

= 1<br />

a1<br />

+ 1<br />

a2<br />

+ · · · + 1<br />

.<br />

an<br />

Let f be a differentiable function on (0, ∞) such that g(x) = f ′<br />

(0, ∞). Then,<br />

Fn = f(x1) + f(x2) + · · · + f(xn)<br />

� 1<br />

√x<br />

�<br />

is strictly convex on<br />

is maximal for 0 < x1 = x2 = · · · = xn−1 ≤ xn, and is minimal for 0 < x1 ≤ x2 = x3 = · · · =<br />

xn.<br />

Corollary 1.7. Let a1, a2, . . . , an (n ≥ 3) be fixed positive numbers, and let 0 < x1 ≤ x2 ≤<br />

· · · ≤ xn such that<br />

x1 + x2 + · · · + xn = a1 + a2 + · · · + an, x1x2 · · · xn = a1a1 · · · an.<br />

Let f be a differentiable function on (0, ∞) such that g(x) = f ′ � �<br />

1 is strictly convex on (0, ∞).<br />

x<br />

Then,<br />

Fn = f(x1) + f(x2) + · · · + f(xn)<br />

is maximal for 0 < x1 = x2 = · · · = xn−1 ≤ xn, and is minimal for 0 < x1 ≤ x2 = x3 = · · · =<br />

xn.<br />

Corollary 1.8. Let a1, a2, . . . , an (n ≥ 3) be fixed non-negative numbers, and let 0 ≤ x1 ≤<br />

x2 ≤ · · · ≤ xn such that<br />

x1 + x2 + · · · + xn = a1 + a2 + · · · + an,<br />

x p<br />

1 + x p<br />

2 + · · · + x p n = a p<br />

1 + a p<br />

2 + · · · + a p n,<br />

where p is a real number, p �= 0 and p �= 1.<br />

(a) For p < 0, P = x1x2 · · · xn is minimal when 0 < x1 = x2 = · · · = xn−1 ≤ xn, and is<br />

maximal when 0 < x1 ≤ x2 = x3 = · · · = xn.<br />

(b) For p > 0, P = x1x2 · · · xn is maximal when 0 ≤ x1 = x2 = · · · = xn−1 ≤ xn, and is<br />

minimal when either x1 = 0 or 0 < x1 ≤ x2 = x3 = · · · = xn.<br />

Corollary 1.9. Let a1, a2, . . . , an (n ≥ 3) be fixed non-negative numbers, let 0 ≤ x1 ≤ x2 ≤<br />

· · · ≤ xn such that<br />

and let E = x q<br />

1 + x q<br />

2 + · · · + x q n.<br />

x1 + x2 + · · · + xn = a1 + a2 + · · · + an,<br />

x p<br />

1 + x p<br />

2 + · · · + x p n = a p<br />

1 + a p<br />

2 + · · · + a p n,<br />

Case 1. p ≤ 0 (p = 0 yields x1x2 · · · xn = a1a2 · · · an > 0).<br />

(a) For q ∈ (p, 0) ∪ (1, ∞), E is maximal when 0 < x1 = x2 = · · · = xn−1 ≤ xn, and is<br />

minimal when 0 < x1 ≤ x2 = x3 = · · · = xn.<br />

(b) For q ∈ (−∞, p) ∪ (0, 1), E is minimal when 0 < x1 = x2 = · · · = xn−1 ≤ xn, and is<br />

maximal when 0 < x1 ≤ x2 = x3 = · · · = xn.<br />

Case 2. 0 < p < 1.<br />

(a) For q ∈ (0, p) ∪ (1, ∞), E is maximal when 0 ≤ x1 = x2 = · · · = xn−1 ≤ xn, and is<br />

minimal when either x1 = 0 or 0 < x1 ≤ x2 = x3 = · · · = xn.<br />

J. Inequal. Pure and Appl. Math., 8(1) (2007), Art. 15, 21 pp. http://jipam.vu.edu.au/


4 VASILE CÎRTOAJE<br />

(b) For q ∈ (−∞, 0) ∪ (p, 1), E is minimal when 0 ≤ x1 = x2 = · · · = xn−1 ≤ xn, and is<br />

maximal when either x1 = 0 or 0 < x1 ≤ x2 = x3 = · · · = xn.<br />

Case 3. p > 1.<br />

(a) For q ∈ (0, 1) ∪ (p, ∞), E is maximal when 0 ≤ x1 = x2 = · · · = xn−1 ≤ xn, and is<br />

minimal when either x1 = 0 or 0 < x1 ≤ x2 = x3 = · · · = xn.<br />

(b) For q ∈ (−∞, 0) ∪ (1, p), E is minimal when 0 ≤ x1 = x2 = · · · = xn−1 ≤ xn, and is<br />

maximal when either x1 = 0 or 0 < x1 ≤ x2 = x3 = · · · = xn.<br />

2. PROOFS<br />

Proof of Lemma 1.1. Let a ≤ b ≤ c. Note that in the excluded cases a = b = c and a = b = 0,<br />

there is a single triple (x, y, z) which verifies the conditions<br />

x + y + z = a + b + c and x p + y p + z p = a p + b p + c p .<br />

Consider now three cases: p = 0, p < 0 and p > 1.<br />

A. Case p = 0 (xyz = abc > 0). Let S = a+b+c and P = 3<br />

3√ abc, where S > P > 0 by AM-GM<br />

Inequality. We have<br />

x + y + z = 3S, xyz = P 3 ,<br />

and from 0 < x ≤ y ≤ z and x < z, it follows that 0 < x < P . Now let<br />

f = y + z − 2 √ yz.<br />

It is clear that f ≥ 0, with equality if and only if y = z. Writing f as a function of x,<br />

�<br />

P<br />

f(x) = 3S − x − 2P<br />

x ,<br />

we have<br />

f ′ (x) = P<br />

�<br />

P<br />

− 1 > 0,<br />

x x<br />

and hence the function f(x) is strictly increasing. Since f(P ) = 3(S − P ) > 0, the equation<br />

f(x) = 0 has a unique positive root x1, 0 < x1 < P . From f(x) ≥ 0, it follows that x ≥ x1.<br />

Sub-case x = x1. Since f(x) = f(x1) = 0 and f = 0 implies y = z, we have 0 < x < y = z.<br />

Sub-case x > x1. We have f(x) > 0 and y < z. Consider now that y and z depend on x. From<br />

x + y(x) + z(x) = 3S and x · y(x) · z(x) = P 3 , we get 1 + y ′ + z ′ = 0 and 1 y′ z′ + + = 0.<br />

x y z<br />

Hence,<br />

y ′ y(x − z)<br />

(x) =<br />

x(z − y) , z′ z(y − x)<br />

(x) =<br />

x(z − y) .<br />

Since y ′ (x) < 0, the function y(x) is strictly decreasing. Since y(x1) > x1 (see sub-case<br />

x = x1), there exists x2 > x1 such that y(x2) = x2, y(x) > x for x1 < x < x2 and y(x) < x<br />

for x > x2. Taking into account that y ≥ x, it follows that x1 < x ≤ x2. On the other hand, we<br />

see that z ′ (x) > 0 for x1 < x < x2. Consequently, the function z(x) is strictly increasing, and<br />

hence z(x) > z(x1) = y(x1) > y(x). Finally, we conclude that x < y < z for x ∈ (x1, x2),<br />

and x = y < z for x = x2.<br />

B. Case p < 0. Denote S = a+b+c<br />

3<br />

and R = � ap +bp +cp � 1<br />

p . Taking into account that<br />

3<br />

x + y + z = 3S, x p + y p + z p = 3R p ,<br />

J. Inequal. Pure and Appl. Math., 8(1) (2007), Art. 15, 21 pp. http://jipam.vu.edu.au/


EQUAL VARIABLE METHOD 5<br />

from 0 < x ≤ y ≤ z and x < z we get x < S and 3 1<br />

p R < x < R. Let<br />

� � −1<br />

p p y + z p<br />

By the AM-GM Inequality, we have<br />

h = (y + z)<br />

h ≥ 2 √ yz 1<br />

√ yz − 2 = 0,<br />

2<br />

− 2.<br />

with equality if and only if y = z. Writing now h as a function of x,<br />

� � −1<br />

p p 3R − x p<br />

h(x) = (3S − x)<br />

− 2,<br />

2<br />

from<br />

h ′ (x) = 3Rp<br />

2<br />

� 3R p − x p<br />

2<br />

� −1−p<br />

p<br />

� �S<br />

it follows that h(x) is strictly increasing. Since h(x) ≥ 0 and h<br />

h(x) = 0 has a unique root x1 and x ≥ x1 > 3 1<br />

p R.<br />

x<br />

� � � �<br />

−p<br />

R<br />

− 1 > 0<br />

x<br />

�<br />

3 1 �<br />

p R<br />

= −2, the equation<br />

Sub-case x = x1. Since f(x) = f(x1) = 0, and f = 0 implies y = z, we have 0 < x < y = z.<br />

Sub-case x > x1. We have h(x) > 0 and y < z. Consider now that y and z depend on x.<br />

From x + y(x) + z(x) = 3S and x p + y(x) p + z(x) p = 3R p , we get 1 + y ′ + z ′ = 0 and<br />

x p−1 + y p−1 y ′ + z p−1 z ′ = 0, and hence<br />

y ′ (x) = xp−1 − zp−1 zp−1 − yp−1 , z′ (x) = xp−1 − yp−1 yp−1 − z<br />

Since y ′ (x) > 0, the function y(x) is strictly decreasing. Since y(x1) > x1 (see sub-case<br />

x = x1), there exists x2 > x1 such that y(x2) = x2, y(x) > x for x1 < x < x2, and y(x) < x for<br />

x > x2. The condition y ≥ x yields x1 < x ≤ x2. We see now that z ′ (x) > 0 for x1 < x < x2.<br />

Consequently, the function z(x) is strictly increasing, and hence z(x) > z(x1) = y(x1) > y(x).<br />

p−1 .<br />

Finally, we have x < y < z for x ∈ (x1, x2) and x = y < z for x = x2.<br />

C. Case p > 1. Denoting S = a+b+c<br />

3<br />

and R = � a p +b p +c p<br />

3<br />

� 1<br />

p yields<br />

x + y + z = 3S, x p + y p + z p = 3R p .<br />

By Jensen’s inequality applied to the convex function g(u) = up , we have R > S, and hence<br />

x < S < R. Let<br />

h = 2<br />

� � 1<br />

p p y + z p<br />

− 1.<br />

y + z 2<br />

By Jensen’s Inequality, we get h ≥ 0, with equality if only if y = z. From<br />

� � 1<br />

p p<br />

2 3R − x p<br />

h(x) =<br />

− 1<br />

3S − x 2<br />

and<br />

h ′ 3<br />

(x) =<br />

(3S − x) 2<br />

� � 1−p<br />

p p 3R − x p<br />

(R<br />

2<br />

p − Sx p−1 ) > 0,<br />

it follows that the function h(x) is strictly increasing, and h(x) ≥ 0 implies x ≥ x1. In the case<br />

h(0) ≥ 0 we have x1 = 0, and in the case h(0) < 0 we have x1 > 0 and h(x1) = 0.<br />

J. Inequal. Pure and Appl. Math., 8(1) (2007), Art. 15, 21 pp. http://jipam.vu.edu.au/


6 VASILE CÎRTOAJE<br />

Sub-case x = x1. If h(0) ≥ 0, then 0 = x1 < y(x1) ≤ z(x1). If h(0) < 0, then h(x1) = 0, and<br />

since h = 0 implies y = z, we have 0 < x1 < y(x1) = z(x1).<br />

Sub-case x > x1. Since h(x) is strictly increasing, for x > x1 we have h(x) > h(x1) ≥ 0,<br />

hence h(x) > 0 and y < z. From x + y(x) + z(x) = 3S and x p + y p (x) + z p (x) = 3R p , we get<br />

y ′ (x) = xp−1 − zp−1 zp−1 − yp−1 , z′ (x) = yp−1 − xp−1 zp−1 − y<br />

Since y ′ (x) < 0, the function y(x) is strictly decreasing. Taking account of y(x1) > x1 (see<br />

sub-case x = x1), there exists x2 > x1 such that y(x2) = x2, y(x) > x for x1 < x < x2,<br />

and y(x) < x for x > x2. The condition y ≥ x implies x1 < x ≤ x2. We see now that<br />

z ′ (x) > 0 for x1 < x < x2. Consequently, the function z(x) is strictly increasing, and hence<br />

z(x) > z(x1) ≥ y(x1) > y(x). Finally, we conclude that x < y < z for x ∈ (x1, x2), and<br />

x = y < z for x = x2. �<br />

Proof of Proposition 1.2. Consider the function<br />

F (x) = f(x) + f(y(x)) + f(z(x))<br />

defined on x ∈ [x1, x2]. We claim that F (x) is minimal for x = x1 and is maximal for x = x2.<br />

If this assertion is true, then by Lemma 1.1 it follows that:<br />

(a) F (x) is minimal for 0 < x = y < z in the case p ≤ 0, or for either x = 0 or<br />

0 < x < y = z in the case p > 1;<br />

(b) F (x) is maximal for 0 < x = y < z.<br />

In order to prove the claim, assume that x ∈ (x1, x2). By Lemma 1.1, we have 0 < x < y <<br />

z. From<br />

we get<br />

whence<br />

x + y(x) + z(x) = a + b + c and<br />

x p + y p (x) + z p (x) = a p + b p + c p ,<br />

p−1 .<br />

y ′ + z ′ = −1, y p−1 y ′ + z p−1 z ′ = −x p−1 ,<br />

y ′ = xp−1 − zp−1 zp−1 − yp−1 , z′ = xp−1 − yp−1 yp−1 − z<br />

It is easy to check that this result is also valid for p = 0. We have<br />

and<br />

F ′ (x) = f ′ (x) + y ′ f ′ (y) + z ′ f ′ (z)<br />

F ′ (x)<br />

(xp−1 − yp−1 )(xp−1 − zp−1 )<br />

g(x<br />

=<br />

p−1 )<br />

(xp−1 − yp−1 )(xp−1 − zp−1 ) +<br />

g(yp−1 )<br />

(yp−1 − zp−1 )(yp−1 − xp−1 )<br />

g(z<br />

+<br />

p−1 )<br />

(zp−1 − xp−1 )(zp−1 − yp−1 ) .<br />

p−1 .<br />

Since g is strictly convex, the right hand side is positive. On the other hand,<br />

(x p−1 − y p−1 )(x p−1 − z p−1 ) > 0.<br />

These results imply F ′ (x) > 0. Consequently, the function F (x) is strictly increasing for<br />

x ∈ (x1, x2). Excepting the trivial case when p > 1, x1 = 0 and lim f(u) = −∞, the function<br />

u→0<br />

J. Inequal. Pure and Appl. Math., 8(1) (2007), Art. 15, 21 pp. http://jipam.vu.edu.au/


EQUAL VARIABLE METHOD 7<br />

F (x) is continuous on [x1, x2], and hence is minimal only for x = x1, and is maximal only for<br />

x = x2. �<br />

Proof of Theorem 1.3. We will consider two cases.<br />

Case p ∈ (−∞, 0]∪(1, ∞). Excepting the trivial case when p > 1, x1 = 0 and lim<br />

u→0 f(u) = −∞,<br />

the function Fn(x1, x2, . . . , xn) attains its minimum and maximum values, and the conclusion<br />

follows from Proposition 1.2 above, via contradiction. For example, let us consider the case p ≤<br />

0. In order to prove that Fn is maximal for 0 < x1 = x2 = · · · = xn−1 ≤ xn, we assume, for the<br />

sake of contradiction, that Fn attains its maximum at (b1, b2, . . . , bn) with b1 ≤ b2 ≤ · · · ≤ bn<br />

and b1 < bn−1. Let x1, xn−1, xn be positive numbers such that x1 + xn−1 + xn = b1 + bn−1 + bn<br />

and x p<br />

1 + x p<br />

n−1 + x p n = b p<br />

1 + b p<br />

n−1 + b p n. According to Proposition 1.2, the expression<br />

F3(x1, xn−1, xn) = f(x1) + f(xn−1) + f(xn)<br />

is maximal only for x1 = xn−1 < xn, which contradicts the assumption that Fn attains its<br />

maximum at (b1, b2, . . . , bn) with b1 < bn−1.<br />

Case p ∈ (0, 1). This case reduces to the case p > 1, replacing each of the ai by a 1<br />

p<br />

i , each of<br />

the xi by x 1<br />

�<br />

p<br />

x 1 �<br />

1−p<br />

i<br />

1<br />

′<br />

, and then p by . Thus, we obtain the sufficient condition that h(x) = xf p<br />

to be strictly convex on (0, ∞). We claim that this condition is equivalent to the condition that<br />

g(x) = f ′<br />

�<br />

x 1 �<br />

p−1 to be strictly convex on (0, ∞). Actually, for our proof, it suffices to show<br />

that if g(x) is strictly convex on (0, ∞), then h(x) is strictly convex on (0, ∞). To show this,<br />

we see that g � �<br />

1 1 = h(x). Since g(x) is strictly convex on (0, ∞), by Jensen’s inequality we<br />

x x<br />

have<br />

� � � �<br />

� u v �<br />

1 1<br />

+ x y<br />

ug + vg > (u + v)g<br />

x y<br />

u + v<br />

for any x, y, u, v > 0 with x �= y. This inequality is equivalent to<br />

� � � �<br />

u v u v u + v<br />

h(x) + h(y) > + h .<br />

x y x y<br />

Substituting u = tx and v = (1 − t)y, where t ∈ (0, 1), reduces the inequality to<br />

u<br />

x<br />

+ v<br />

y<br />

th(x) + (1 − t)h(y) > h(tx + (1 − t)y),<br />

which shows us that h(x) is strictly convex on (0, ∞). �<br />

Proof of Corollary 1.8. We will apply Theorem 1.3 to the function f(u) = p ln u. We see that<br />

lim f(u) = −∞ for p > 0, and<br />

u→0<br />

f ′ (u) = p<br />

�<br />

′<br />

, g(x) = f x<br />

u 1 �<br />

p−1 = px 1<br />

1−p , g ′′ (x) =<br />

p2 2p−1<br />

x 1−p .<br />

(1 − p) 2<br />

Since g ′′ (x) > 0 for x > 0, the function g(x) is strictly convex on (0, ∞), and the conclusion<br />

follows by Theorem 1.3. �<br />

Proof of Corollary 1.9. We will apply Theorem 1.3 to the function<br />

f(u) = q(q − 1)(q − p)u q .<br />

For p > 0, it is easy to check that either f(u) is continuous at u = 0 (in the case q > 0) or<br />

lim f(u) = −∞ (in the case q < 0). We have<br />

u→0<br />

f ′ (u) = q 2 (q − 1)(q − p)u q−1<br />

J. Inequal. Pure and Appl. Math., 8(1) (2007), Art. 15, 21 pp. http://jipam.vu.edu.au/


8 VASILE CÎRTOAJE<br />

and<br />

g(x) = f ′<br />

�<br />

x 1 �<br />

p−1 = q 2 (q − 1)(q − p)x q−1<br />

p−1 ,<br />

g ′′ (x) = q2 (q − 1) 2 (q − p) 2<br />

(p − 1) 2 x 2p−1<br />

1−p .<br />

Since g ′′ (x) > 0 for x > 0, the function g(x) is strictly convex on (0, ∞), and the conclusion<br />

follows by Theorem 1.3. �<br />

3. APPLICATIONS<br />

Proposition 3.1. Let x, y, z be non-negative real numbers such that x+y+z = 2. If r0 ≤ r ≤ 3,<br />

where r0 = ≈ 1.71, then<br />

ln 2<br />

ln 3−ln 2<br />

x r (y + z) + y r (z + x) + z r (x + y) ≤ 2.<br />

Proof. Rewrite the inequality in the homogeneous form<br />

x r+1 + y r+1 + z r+1 � �r+1 x + y + z<br />

+ 2<br />

2<br />

and apply Corollary 1.9 (case p = r and q = r + 1):<br />

If 0 ≤ x ≤ y ≤ z such that<br />

x + y + z = constant and<br />

x r + y r + z r = constant,<br />

≥ (x + y + z)(x r + y r + z r ),<br />

then the sum x r+1 + y r+1 + z r+1 is minimal when either x = 0 or 0 < x ≤ y = z.<br />

Case x = 0. The initial inequality becomes<br />

yz(y r−1 + z r−1 ) ≤ 2,<br />

where y + z = 2. Since 0 < r − 1 ≤ 2, by the Power Mean inequality we have<br />

yr−1 + zr−1 � � r−1<br />

2 2 2 y + z<br />

≤<br />

.<br />

2<br />

2<br />

Thus, it suffices to show that<br />

Taking account of<br />

� � r−1<br />

2 2 2 y + z<br />

yz<br />

≤ 1.<br />

2<br />

y2 + z2 =<br />

2<br />

2(y2 + z2 )<br />

≥ 1<br />

(y + z) 2 and<br />

r − 1<br />

≤ 1,<br />

2<br />

we have<br />

� � r−1 � �<br />

2 2 2<br />

2 2<br />

y + z y + z<br />

1 − yz<br />

≥ 1 − yz<br />

2<br />

2<br />

= (y + z)4<br />

16<br />

− yz(y2 + z2 )<br />

2<br />

= (y − z)4<br />

≥ 0.<br />

16<br />

Case 0 < x ≤ y = z. In the homogeneous inequality we may leave aside the constraint<br />

x + y + z = 2, and consider y = z = 1, 0 < x ≤ 1. The inequality reduces to<br />

�<br />

1 + x<br />

�r+1 − x<br />

2<br />

r − x − 1 ≥ 0.<br />

J. Inequal. Pure and Appl. Math., 8(1) (2007), Art. 15, 21 pp. http://jipam.vu.edu.au/


0<br />

EQUAL VARIABLE METHOD 9<br />

Since � 1 + x<br />

�r+1 r is increasing and x is decreasing in respect to r, it suffices to consider r = r0.<br />

2<br />

Let<br />

�<br />

f(x) = 1 + x<br />

�r0+1 − x<br />

2<br />

r0 − x − 1.<br />

We have<br />

f ′ (x) = r0 + 1<br />

�<br />

1 +<br />

2<br />

x<br />

�r0 − r0x<br />

2<br />

r0−1<br />

− 1,<br />

1<br />

f ′′ (x) = r0 + 1<br />

�<br />

1 +<br />

4<br />

x<br />

�r0 −<br />

2<br />

r0 − 1<br />

. 2−r0 x<br />

Since f ′′ (x) is strictly increasing on (0, 1], f ′′ (0+) = −∞ and<br />

1<br />

f ′′ (1) = r0<br />

� �r0 + 1 3<br />

− r0 + 1<br />

4 2<br />

r0<br />

= r0 + 1<br />

2 − r0 + 1 =<br />

3 − r0<br />

2<br />

there exists x1 ∈ (0, 1) such that f ′′ (x1) = 0, f ′′ (x) < 0 for x ∈ (0, x1), and f ′′ (x) > 0 for<br />

x ∈ (x1, 1]. Therefore, the function f ′ (x) is strictly decreasing for x ∈ [0, x1], and strictly<br />

increasing for x ∈ [x1, 1]. Since<br />

f ′ (0) = r0 − 1<br />

2 > 0 and f ′ (1) = r0 + 1<br />

2<br />

> 0,<br />

�� �r0 �<br />

3<br />

− 2 = 0,<br />

2<br />

there exists x2 ∈ (0, x1) such that f ′ (x2) = 0, f ′ (x) > 0 for x ∈ [0, x2), and f ′ (x) < 0 for<br />

x ∈ (x2, 1). Thus, the function f(x) is strictly increasing for x ∈ [0, x2], and strictly decreasing<br />

for x ∈ [x2, 1]. Since f(0) = f(1) = 0, it follows that f(x) ≥ 0 for 0 < x ≤ 1, establishing the<br />

desired result.<br />

For x ≤ y ≤ z, equality occurs when x = 0 and y = z = 1. Moreover, for r = r0, equality<br />

holds again when x = y = z = 1. �<br />

Proposition 3.2 ([12]). Let x, y, z be non-negative real numbers such that xy + yz + zx = 3.<br />

If 1 < r ≤ 2, then<br />

x r (y + z) + y r (z + x) + z r (x + y) ≥ 6.<br />

Proof. Rewrite the inequality in the homogeneous form<br />

x r (y + z) + y r (z + x) + z r �<br />

xy + yz + zx<br />

(x + y) ≥ 6<br />

3<br />

� r+1<br />

2<br />

For convenience, we may leave aside the constraint xy +yz +zx = 3. Using now the constraint<br />

x + y + z = 1, the inequality becomes<br />

x r (1 − x) + y r (1 − y) + z r � � r+1<br />

2 2 2 2<br />

1 − x − y − z<br />

(1 − z) ≥ 6<br />

.<br />

6<br />

To prove it, we will apply Corollary 1.5 to the function f(u) = −u r (1 − u) for 0 ≤ u ≤ 1. We<br />

have f ′ (u) = −ru r−1 + (r + 1)u r and<br />

g(x) = f ′ (x) = −rx r−1 + (r + 1)x r , g ′′ (x) = r(r − 1)x r−3 [(r + 1)x + 2 − r].<br />

Since g ′′ (x) > 0 for x > 0, g(x) is strictly convex on [0, ∞). According to Corollary 1.5,<br />

if 0 ≤ x ≤ y ≤ z such that x + y + z = 1 and x 2 + y 2 + z 2 = constant, then the sum<br />

f(x) + f(y) + f(z) is maximal for 0 ≤ x = y ≤ z.<br />

J. Inequal. Pure and Appl. Math., 8(1) (2007), Art. 15, 21 pp. http://jipam.vu.edu.au/<br />

.


10 VASILE CÎRTOAJE<br />

Thus, we have only to prove the original inequality in the case x = y ≤ z. This means, to<br />

prove that 0 < x ≤ 1 ≤ y and x 2 + 2xz = 3 implies<br />

x r (x + z) + xz r ≥ 3.<br />

Let f(x) = xr (x + z) + xzr − 3, with z = 3−x2<br />

2x .<br />

Differentiating the equation x2 + 2xz = 3 yields z ′ = −(x+z)<br />

x<br />

. Then,<br />

f ′ (x) = (r + 1)x r + rx r−1 z + z r + (x r + rxz r−1 )z ′<br />

= (x r−1 − z r−1 )[rx + (r − 1)z] ≤ 0.<br />

The function f(x) is strictly decreasing on [0, 1], and hence f(x) ≥ f(1) = 0 for 0 < x ≤ 1.<br />

Equality occurs if and only if x = y = z = 1. �<br />

Proposition 3.3 ([5]). If x1, x2, . . . , xn are positive real numbers such that<br />

then<br />

x1 + x2 + · · · + xn = 1<br />

x1<br />

+ 1<br />

+ · · · +<br />

x2<br />

1<br />

,<br />

xn<br />

1<br />

1<br />

1<br />

+<br />

+ · · · +<br />

≥ 1.<br />

1 + (n − 1)x1 1 + (n − 1)x2 1 + (n − 1)xn<br />

Proof. We have to consider two cases.<br />

Case n = 2. The inequality is verified as equality.<br />

Case n ≥ 3. Assume that 0 < x1 ≤ x2 ≤ · · · ≤ xn, and then apply Corollary 1.6 to the function<br />

1<br />

f(u) = 1+(n−1)u for u > 0. We have f ′ (u) = −(n−1)<br />

[1+(n−1)u] 2 and<br />

g(x) = f ′<br />

� �<br />

1<br />

√x = −(n − 1)x<br />

( √ 2 ,<br />

x + n − 1)<br />

g ′′ 3(n − 1)<br />

(x) =<br />

2<br />

2 √ x ( √ 4 .<br />

x + n − 1)<br />

Since g ′′ (x) > 0, g(x) is strictly convex on (0, ∞). According to Corollary 1.6, if 0 < x1 ≤<br />

x2 ≤ · · · ≤ xn such that<br />

1<br />

x1<br />

x1 + x2 + · · · + xn = constant and<br />

+ 1<br />

+ · · · +<br />

x2<br />

1<br />

xn<br />

= constant,<br />

then the sum f(x1) + f(x2) + · · · + f(xn) is minimal when 0 < x1 ≤ x2 = x3 = · · · = xn.<br />

Thus, we have to prove the inequality<br />

under the constraints 0 < x ≤ 1 ≤ y and<br />

The last constraint is equivalent to<br />

1<br />

1 + (n − 1)x +<br />

n − 1<br />

1 + (n − 1)y<br />

x + (n − 1)y = 1<br />

x<br />

n − 1<br />

+ .<br />

y<br />

(n − 1)(y − 1) = y(1 − x2 )<br />

x(1 + y) .<br />

J. Inequal. Pure and Appl. Math., 8(1) (2007), Art. 15, 21 pp. http://jipam.vu.edu.au/<br />

≥ 1,


Since<br />

we must show that<br />

EQUAL VARIABLE METHOD 11<br />

1<br />

1 + (n − 1)x +<br />

n − 1<br />

− 1<br />

1 + (n − 1)y<br />

1 1<br />

=<br />

−<br />

1 + (n − 1)x n +<br />

n − 1 n − 1<br />

−<br />

1 + (n − 1)y n<br />

= (n − 1)(1 − x)<br />

n[1 + (n − 1)x] − (n − 1)2 (y − 1)<br />

n[1 + (n − 1)y]<br />

= (n − 1)(1 − x)<br />

n[1 + (n − 1)x] −<br />

(n − 1)y(1 − x2 )<br />

nx(1 + y)[1 + (n − 1)y] ,<br />

x(1 + y)[1 + (n − 1)y] ≥ y(1 + x)[1 + (n − 1)x],<br />

which reduces to<br />

(y − x)[(n − 1)xy − 1] ≥ 0.<br />

Since y − x ≥ 0, we have still to prove that<br />

Indeed, from x + (n − 1)y = 1 n−1 + x y<br />

(n − 1)xy − 1 =<br />

(n − 1)xy ≥ 1.<br />

y+(n−1)x<br />

we get xy = , and hence<br />

x+(n−1)y<br />

n(n − 2)x<br />

x + (n − 1)y<br />

For n ≥ 3, one has equality if and only if x1 = x2 = · · · = xn = 1. �<br />

Proposition 3.4 ([10]). Let a1, a2, . . . , an be positive real numbers such that a1a2 · · · an = 1. If<br />

m is a positive integer satisfying m ≥ n − 1, then<br />

a m 1 + a m 2 + · · · + a m �<br />

1<br />

n + (m − 1)n ≥ m +<br />

a1<br />

1<br />

+ · · · +<br />

a2<br />

1<br />

�<br />

.<br />

an<br />

Proof. For n = 2 (hence m ≥ 1), the inequality reduces to<br />

> 0.<br />

a m 1 + a m 2 + 2m − 2 ≥ m(a1 + a2).<br />

We can prove it by summing the inequalities a m 1 ≥ 1+m(a1−1) and a m 2 ≥ 1+m(a2−1), which<br />

are straightforward consequences of Bernoulli’s inequality. For n ≥ 3, replacing a1, a2, . . . , an<br />

by 1 1 1 , , . . . , , respectively, we have to show that<br />

x1 x2 xn<br />

1<br />

+ (m − 1)n ≥ m(x1 + x2 + · · · + xn)<br />

x m 1<br />

+ 1<br />

x m 2<br />

+ · · · + 1<br />

x m n<br />

for x1x2 · · · xn = 1. Assume 0 < x1 ≤ x2 ≤ · · · ≤ xn and apply Corollary 1.9 (case p = 0 and<br />

q = −m):<br />

If 0 < x1 ≤ x2 ≤ · · · ≤ xn such that<br />

then the sum 1<br />

x m 1<br />

+ 1<br />

x m 2<br />

+ · · · + 1<br />

x m n<br />

x1 + x2 + · · · + xn = constant and<br />

x1x2 · · · xn = 1,<br />

is minimal when 0 < x1 = x2 = · · · = xn−1 ≤ xn.<br />

Thus, it suffices to prove the inequality for x1 = x2 = · · · = xn−1 = x ≤ 1, xn = y and<br />

x n−1 y = 1, when it reduces to:<br />

n − 1 1<br />

+ + (m − 1)n ≥ m(n − 1)x + my.<br />

xm ym J. Inequal. Pure and Appl. Math., 8(1) (2007), Art. 15, 21 pp. http://jipam.vu.edu.au/


12 VASILE CÎRTOAJE<br />

By the AM-GM inequality, we have<br />

n − 1<br />

m<br />

+ (m − n + 1) ≥ = my.<br />

xm xn−1 Then, we have still to show that<br />

1<br />

− 1 ≥ m(n − 1)(x − 1).<br />

ym This inequality is equivalent to<br />

and<br />

x mn−m − 1 − m(n − 1)(x − 1) ≥ 0<br />

(x − 1)[(x mn−m−1 − 1) + (x mn−m−2 − 1) + · · · + (x − 1)] ≥ 0.<br />

The last inequality is clearly true. For n = 2 and m = 1, the inequality becomes equality.<br />

Otherwise, equality occurs if and only if a1 = a2 = · · · = an = 1. �<br />

Proposition 3.5 ([6]). Let x1, x2, . . . , xn be non-negative real numbers such that x1 +x2 +· · ·+<br />

xn = n. If k is a positive integer satisfying 2 ≤ k ≤ n + 2, and r = � �<br />

n k−1<br />

− 1, then<br />

n−1<br />

x k 1 + x k 2 + · · · + x k n − n ≥ nr(1 − x1x2 · · · xn).<br />

Proof. If n = 2, then the inequality reduces to x k 1 + x k 2 − 2 ≥ (2 k − 2)x1x2. For k = 2 and<br />

k = 3, this inequality becomes equality, while for k = 4 it reduces to 6x1x2(1 − x1x2) ≥ 0,<br />

which is clearly true.<br />

Consider now n ≥ 3 and 0 ≤ x1 ≤ x2 ≤ · · · ≤ xn. Towards proving the inequality,<br />

we will apply Corollary 1.8 (case p = k > 0): If 0 ≤ x1 ≤ x2 ≤ · · · ≤ xn such that<br />

x1 + x2 + · · · + xn = n and x k 1 + x k 2 + · · · + x k n = constant, then the product x1x2 · · · xn is<br />

minimal when either x1 = 0 or 0 < x1 ≤ x2 = x3 = · · · = xn.<br />

Case x1 = 0. The inequality reduces to<br />

x k 2 + · · · + x k n ≥<br />

nk ,<br />

(n − 1) k−1<br />

with x2 + · · · + xn = n, This inequality follows by applying Jensen’s inequality to the convex<br />

function f(u) = uk :<br />

x k 2 + · · · + x k � �k x2 + · · · + xn<br />

n ≥ (n − 1)<br />

.<br />

n − 1<br />

Case 0 < x1 ≤ x2 = x3 = · · · = xn. Denoting x1 = x and x2 = x3 = · · · = xn = y, we have<br />

to prove that for 0 < x ≤ 1 ≤ y and x + (n − 1)y = n, the inequality holds:<br />

Write the inequality as f(x) ≥ 0, where<br />

x k + (n − 1)y k + nrxy n−1 − n(r + 1) ≥ 0.<br />

f(x) = x k + (n − 1)y k + nrxy n−1 − n(r + 1), with y =<br />

We see that f(0) = f(1) = 0. Since y ′ = −1 , we have<br />

n−1<br />

f ′ (x) = k(x k−1 − y k−1 ) + nry n−2 (y − x)<br />

= (y − x)[nry n−2 − k(y k−2 + y k−3 x + · · · + x k−2 )]<br />

= (y − x)y n−2 [nr − kg(x)],<br />

n − x<br />

n − 1 .<br />

J. Inequal. Pure and Appl. Math., 8(1) (2007), Art. 15, 21 pp. http://jipam.vu.edu.au/


where<br />

EQUAL VARIABLE METHOD 13<br />

g(x) = 1 x xk−2<br />

+ + · · · + .<br />

yn−k yn−k+1 yn−2 Since the function y(x) = n−x is strictly decreasing, the function g(x) is strictly increasing for<br />

n−1<br />

2 ≤ k ≤ n. For k = n + 1, we have<br />

and for k = n + 2, we have<br />

g(x) = y + x + x2 xn−1<br />

+ · · · +<br />

y yn−2 = (n − 2)x + n<br />

+<br />

n − 1<br />

x2 xn−1<br />

+ · · · + ,<br />

y yn−2 g(x) = y 2 + yx + x 2 + x3<br />

y<br />

+ · · · + xn<br />

y n−2<br />

= (n2 − 3n + 3)x 2 + n(n − 3)x + n 2<br />

(n − 1) 2<br />

+ x3<br />

y<br />

xn<br />

+ · · · + .<br />

yn−2 Therefore, the function g(x) is strictly increasing for 2 ≤ k ≤ n + 2, and the function<br />

is strictly decreasing. Note that<br />

h(x) = nr − kg(x)<br />

f ′ (x) = (y − x)y n−2 h(x).<br />

We assert that h(0) > 0 and h(1) < 0. If our claim is true, then there exists x1 ∈ (0, 1) such that<br />

h(x1) = 0, h(x) > 0 for x ∈ [0, x1), and h(x) < 0 for x ∈ (x1, 1]. Consequently, f(x) is strictly<br />

increasing for x ∈ [0, x1], and strictly decreasing for x ∈ [x1, 1]. Since f(0) = f(1) = 0, it<br />

follows that f(x) ≥ 0 for 0 < x ≤ 1, and the proof is completed.<br />

In order to prove that h(0) > 0, we assume that h(0) ≤ 0. Then, h(x) < 0 for x ∈ (0, 1),<br />

f ′ (x) < 0 for x ∈ (0, 1), and f(x) is strictly decreasing for x ∈ [0, 1], which contradicts<br />

f(0) = f(1). Also, if h(1) ≥ 0, then h(x) > 0 for x ∈ (0, 1), f ′ (x) > 0 for x ∈ (0, 1), and<br />

f(x) is strictly increasing for x ∈ [0, 1], which also contradicts f(0) = f(1).<br />

For n ≥ 3 and x1 ≤ x2 ≤ · · · ≤ xn, equality occurs when x1 = x2 = · · · = xn = 1, and also<br />

. �<br />

when x1 = 0 and x2 = · · · = xn = n<br />

n−1<br />

Remark 3.6. For k = 2, k = 3 and k = 4, we get the following nice inequalities:<br />

(n − 1)(x 2 1 + x 2 2 + · · · + x 2 n) + nx1x2 · · · xn ≥ n 2 ,<br />

(n − 1) 2 (x 3 1 + x 3 2 + · · · + x 3 n) + n(2n − 1)x1x2 · · · xn ≥ n 3 ,<br />

(n − 1) 3 (x 4 1 + x 4 2 + · · · + x 4 n) + n(3n 2 − 3n + 1)x1x2 · · · xn ≥ n 4 .<br />

Remark 3.7. The inequality for k = n was posted in 2004 on the Mathlinks Site - Inequalities<br />

Forum by Gabriel Dospinescu and Călin Popa.<br />

Proposition 3.8 ([11]). Let x1, x2, . . . , xn be positive real numbers such that 1<br />

x1<br />

n. Then<br />

where en−1 = � 1 + 1<br />

�n−1 < e.<br />

n−1<br />

x1 + x2 + · · · + xn − n ≤ en−1(x1x2 · · · xn − 1),<br />

1<br />

1<br />

+ +· · ·+ x2 xn =<br />

J. Inequal. Pure and Appl. Math., 8(1) (2007), Art. 15, 21 pp. http://jipam.vu.edu.au/


14 VASILE CÎRTOAJE<br />

Proof. Replacing each of the xi by 1 , the statement becomes as follows:<br />

ai<br />

If a1, a2, . . . , an are positive numbers such that a1 + a2 + · · · + an = n, then<br />

� 1<br />

a1a2 · · · an +<br />

a1<br />

1<br />

a2<br />

+ · · · + 1<br />

an<br />

− n + en−1 ≤ en−1.<br />

It is easy to check that the inequality holds for n = 2. Consider now n ≥ 3, assume that<br />

0 < a1 ≤ a2 ≤ · · · ≤ an and apply Corollary 1.8 (case p = −1): If 0 < a1 ≤ a2 ≤ · · · ≤ an<br />

such that a1 + a2 + · · · + an = n and 1<br />

a1<br />

+ 1<br />

a2<br />

�<br />

+ · · · + 1<br />

an = constant, then the product a1a2 · · · an<br />

is maximal when 0 < a1 ≤ a2 = a3 = · · · = an.<br />

Denoting a1 = x and a2 = a3 = · · · = an = y, we have to prove that for 0 < x ≤ 1 ≤ y <<br />

and x + (n − 1)y = n, the inequality holds:<br />

n<br />

n−1<br />

Letting<br />

y n−1 + (n − 1)xy n−2 − (n − en−1)xy n−1 ≤ en−1.<br />

f(x) = y n−1 + (n − 1)xy n−2 − (n − en−1)xy n−1 − en−1, with<br />

y =<br />

n − x<br />

n − 1 ,<br />

we must show that f(x) ≤ 0 for 0 < x ≤ 1. We see that f(0) = f(1) = 0. Since y ′ = −1<br />

n−1<br />

have<br />

where<br />

f ′ (x)<br />

y n−3 = (y − x)[n − 2 − (n − en−1)y] = (y − x)h(x),<br />

n − x<br />

h(x) = n − 2 − (n − en−1)<br />

n − 1<br />

is a linear increasing function.<br />

Let us show that h(0) < 0 and h(1) > 0. If h(0) ≥ 0, then h(x) > 0 for x ∈ (0, 1),<br />

hence f ′ (x) > 0 for x ∈ (0, 1), and f(x) is strictly increasing for x ∈ [0, 1], which contradicts<br />

f(0) = f(1). Also, h(1) = en−1 − 2 > 0.<br />

From h(0) < 0 and h(1) > 0, it follows that there exists x1 ∈ (0, 1) such that h(x1) = 0,<br />

h(x) < 0 for x ∈ [0, x1), and h(x) > 0 for x ∈ (x1, 1]. Consequently, f(x) is strictly decreasing<br />

for x ∈ [0, x1], and strictly increasing for x ∈ [x1, 1]. Since f(0) = f(1) = 0, it follows that<br />

f(x) ≤ 0 for 0 ≤ x ≤ 1.<br />

For n ≥ 3, equality occurs when x1 = x2 = · · · = xn = 1. �<br />

Proposition 3.9 ([9]). If x1, x2, . . . , xn are positive real numbers, then<br />

x n 1 + x n 2 + · · · + x n n + n(n − 1)x1x2 · · · xn<br />

�<br />

1<br />

≥ x1x2 · · · xn(x1 + x2 + · · · + xn)<br />

x1<br />

, we<br />

+ 1<br />

+ · · · +<br />

x2<br />

1<br />

�<br />

.<br />

xn<br />

Proof. For n = 2, one has equality. Assume now that n ≥ 3, 0 < x1 ≤ x2 ≤ · · · ≤ xn and<br />

apply Corollary 1.9 (case p = 0): If 0 < x1 ≤ x2 ≤ · · · ≤ xn such that<br />

x1 + x2 + · · · + xn = constant and<br />

x1x2 · · · xn = constant,<br />

then the sum x n 1 + x n 2 + · · · + x n n is minimal and the sum 1<br />

x1<br />

1<br />

1<br />

+ + · · · + x2 xn<br />

is maximal when<br />

0 < x1 ≤ x2 = x3 = · · · = xn.<br />

Thus, it suffices to prove the inequality for 0 < x1 ≤ 1 and x2 = x3 = · · · = xn = 1. The<br />

inequality becomes<br />

x n 1 + (n − 2)x1 ≥ (n − 1)x 2 1,<br />

J. Inequal. Pure and Appl. Math., 8(1) (2007), Art. 15, 21 pp. http://jipam.vu.edu.au/


and is equivalent to<br />

x1(x1 − 1)[(x n−2<br />

1<br />

EQUAL VARIABLE METHOD 15<br />

− 1) + (x n−3<br />

1<br />

− 1) + · · · + (x1 − 1)] ≥ 0,<br />

which is clearly true. For n ≥ 3, equality occurs if and only if x1 = x2 = · · · = xn. �<br />

Proposition 3.10 ([14]). If x1, x2, . . . , xn are non-negative real numbers, then<br />

(n − 1)(x n 1 + x n 2 + · · · + x n n) + nx1x2 · · · xn<br />

≥ (x1 + x2 + · · · + xn)(x n−1<br />

1<br />

+ x n−1<br />

2<br />

+ · · · + x n−1<br />

n ).<br />

Proof. For n = 2, one has equality. For n ≥ 3, assume that 0 ≤ x1 ≤ x2 ≤ · · · ≤ xn<br />

and apply Corollary 1.9 (case p = n and q = n − 1) and Corollary 1.8 (case p = n): If<br />

0 ≤ x1 ≤ x2 ≤ · · · ≤ xn such that<br />

then the sum x n−1<br />

1<br />

+ x n−1<br />

2<br />

x1 + x2 + · · · + xn = constant and<br />

x n 1 + x n 2 + · · · + x n n = constant,<br />

+ · · · + x n−1<br />

n<br />

is maximal and the product x1x2 · · · xn is minimal when<br />

either x1 = 0 or 0 < x1 ≤ x2 = x3 = · · · = xn.<br />

So, it suffices to consider the cases x1 = 0 and 0 < x1 ≤ x2 = x3 = · · · = xn.<br />

Case x1 = 0. The inequality reduces to<br />

(n − 1)(x n 2 + · · · + x n n) ≥ (x2 + · · · + xn)(x n−1<br />

2<br />

+ · · · + x n−1<br />

n ),<br />

which immediately follows by Chebyshev’s inequality.<br />

Case 0 < x1 ≤ x2 = x3 = · · · = xn. Setting x2 = x3 = · · · = xn = 1, the inequality reduces<br />

to:<br />

Rewriting this inequality as<br />

x1(x1 − 1)[x n−3<br />

1<br />

(n − 2)x n 1 + x1 ≥ (n − 1)x n−1<br />

1 .<br />

(x1 − 1) + x n−4<br />

1<br />

(x 2 1 − 1) + · · · + (x n−2<br />

1<br />

− 1)] ≥ 0,<br />

we see that it is clearly true. For n ≥ 3 and x1 ≤ x2 ≤ · · · ≤ xn equality occurs when<br />

x1 = x2 = · · · = xn, and for x1 = 0 and x2 = · · · = xn. �<br />

Proposition 3.11 ([8]). If x1, x2, . . . , xn are positive real numbers, then<br />

�<br />

1<br />

(x1 + x2 + · · · + xn − n) +<br />

x1<br />

1<br />

+ · · · +<br />

x2<br />

1<br />

�<br />

− n + x1x2 · · · xn +<br />

xn<br />

Proof. For n = 2, the inequality reduces to<br />

(1 − x1) 2 (1 − x2) 2<br />

x1x2<br />

≥ 0.<br />

1<br />

x1x2 · · · xn<br />

For n ≥ 3, assume that 0 < x1 ≤ x2 ≤ · · · ≤ xn. Since the inequality preserves its form<br />

by replacing each number xi with 1<br />

xi , we may consider x1x2 · · · xn ≥ 1. So, by the AM-GM<br />

inequality we get<br />

x1 + x2 + · · · + xn − n ≥ n n√ x1x2 · · · xn − n ≥ 0,<br />

and we may apply Corollary 1.9 (case p = 0 and q = −1): If 0 ≤ x1 ≤ x2 ≤ · · · ≤ xn such<br />

that<br />

x1 + x2 + · · · + xn = constant and<br />

x1x2 · · · xn = constant,<br />

then the sum 1 1<br />

1<br />

+ + · · · + x1 x2 xn is minimal when 0 < x1 = x2 = · · · = xn−1 ≤ xn.<br />

J. Inequal. Pure and Appl. Math., 8(1) (2007), Art. 15, 21 pp. http://jipam.vu.edu.au/<br />

≥ 2.


16 VASILE CÎRTOAJE<br />

According to this statement, it suffices to consider x1 = x2 = · · · = xn−1 = x and xn = y,<br />

when the inequality reduces to<br />

� �<br />

n − 1 1<br />

((n − 1)x + y − n) + − n + x<br />

x y n−1 y + 1<br />

xn−1 ≥ 2,<br />

y<br />

or �<br />

x n−1 � � �<br />

n − 1<br />

1<br />

1 n(n − 1)(x − 1)2<br />

+ − n y + + (n − 1)x − n ≥ .<br />

x xn−1 y x<br />

Since<br />

x n−1 n − 1 x − 1<br />

+ − n =<br />

x x [(xn−1 − 1) + (x n−2 − 1) + · · · + (x − 1)]<br />

= (x − 1)2<br />

[x<br />

x<br />

n−2 + 2x n−3 + · · · + (n − 1)]<br />

and<br />

� �<br />

1<br />

(x − 1)2 1 2<br />

+ (n − 1)x − n = + + · · · + (n − 1) ,<br />

xn−1 x xn−2 xn−3 it is enough to show that<br />

[x n−2 + 2x n−3 � �<br />

1 2<br />

1<br />

+ · · · + (n − 1)]y + + + · · · + (n − 1) ≥ n(n − 1).<br />

xn−2 xn−3 y<br />

This inequality is equivalent to<br />

�<br />

x n−2 y + 1<br />

xn−2 �<br />

− 2<br />

y<br />

�<br />

+ 2 x n−3 y + 1<br />

xn−3 �<br />

− 2<br />

y<br />

+ · · · + (n − 1)<br />

�<br />

y + 1<br />

�<br />

− 2 ≥ 0,<br />

y<br />

or<br />

(xn−2y − 1) 2<br />

xn−2 +<br />

y<br />

2(xn−3y − 1) 2<br />

xn−3 (n − 1)(y − 1)2<br />

+ · · · + ≥ 0,<br />

y<br />

y<br />

which is clearly true. Equality occurs if and only if n − 1 of the numbers xi are equal to 1. �<br />

Proposition 3.12 ([15]). If x1, x2, . . . , xn are non-negative real numbers such that x1 + x2 +<br />

· · · + xn = n, then<br />

(x1x2 · · · xn)<br />

√<br />

1<br />

n−1 (x 2 1 + x 2 2 + · · · + x 2 n) ≤ n.<br />

Proof. For n = 2, the inequality reduces to 2(x1x2 − 1) 2 ≥ 0. For n ≥ 3, assume that<br />

0 ≤ x1 ≤ x2 ≤ · · · ≤ xn and apply Corollary 1.8 (case p = 2): If 0 ≤ x1 ≤ x2 ≤ · · · ≤ xn<br />

such that x1 +x2 +· · ·+xn = n and x 2 1 +x 2 2 +· · ·+x 2 n = constant, then the product x1x2 · · · xn<br />

is maximal when 0 ≤ x1 = x2 = · · · = xn−1 ≤ xn.<br />

Consequently, it suffices to show that the inequality holds for x1 = x2 = · · · = xn−1 = x and<br />

xn = y, where 0 ≤ x ≤ 1 ≤ y and (n − 1)x + y = n. Under the circumstances, the inequality<br />

reduces to<br />

x √ n−1 y 1<br />

√n−1 [(n − 1)x 2 + y 2 ] ≤ n.<br />

For x = 0, the inequality is trivial. For x > 0, it is equivalent to f(x) ≤ 0, where<br />

f(x) = √ n − 1 ln x +<br />

1<br />

√ n − 1 ln y + ln[(n − 1)x 2 + y 2 ] − ln n,<br />

with y = n − (n − 1)x.<br />

J. Inequal. Pure and Appl. Math., 8(1) (2007), Art. 15, 21 pp. http://jipam.vu.edu.au/


We have y ′ = −(n − 1) and<br />

EQUAL VARIABLE METHOD 17<br />

f ′ (x)<br />

√ =<br />

n − 1 1 1<br />

−<br />

x y + 2√n − 1(x − y)<br />

(n − 1)x2 + y2 = (y − x)(√n − 1x − y) 2<br />

xy[(n − 1)x2 + y2 ]<br />

Therefore, the function f(x) is strictly increasing on (0, 1] and hence f(x) ≤ f(1) = 0. Equality<br />

occurs if and only if x1 = x2 = · · · = xn = 1. �<br />

≥ 0.<br />

Remark 3.13. For n = 5, we get the following nice statement:<br />

If a, b, c, d, e are positive real numbers such that a 2 + b 2 + c 2 + d 2 + e 2 = 5, then<br />

abcde(a 4 + b 4 + c 4 + d 4 + e 4 ) ≤ 5.<br />

Proposition 3.14 ([4]). Let x, y, z be non-negative real numbers such that xy + yz + zx = 3,<br />

and let<br />

ln 9 − ln 4<br />

p ≥ ≈ 0.738.<br />

ln 3<br />

Then,<br />

Proof. Let r =<br />

Thus, it suffices to show that<br />

x p + y p + z p ≥ 3.<br />

ln 9−ln 4.<br />

By the Power-Mean inequality, we have<br />

ln 3<br />

xp + yp + zp � � p<br />

r r r<br />

x + y + z r<br />

≥<br />

.<br />

3<br />

3<br />

x r + y r + z r ≥ 3.<br />

Let x ≤ y ≤ z. We consider two cases.<br />

Case x = 0. We have to show that y r + z r ≥ 3 for yz = 3. Indeed, by the AM-GM inequality,<br />

we get<br />

y r + z r ≥ 2(yz) r/2 = 2 · 3 r/2 = 3.<br />

Case x > 0. The inequality x r + y r + z r ≥ 3 is equivalent to the homogeneous inequality<br />

x r + y r + z r ≥ 3<br />

� xyz<br />

3<br />

� r<br />

2<br />

�<br />

1 1 1<br />

+ +<br />

x y z<br />

� r<br />

2<br />

.<br />

Setting x = a 1<br />

r , y = b 1<br />

r , z = c 1<br />

r (0 < a ≤ b ≤ c), the inequality becomes<br />

� � 1<br />

abc 2 �<br />

a + b + c ≥ 3 a −1<br />

r + b −1<br />

r + c −1<br />

� r<br />

2<br />

r .<br />

3<br />

Towards proving this inequality, we apply Corollary 1.9 (case p = 0, q = −1):<br />

If 0 < a ≤ b ≤ c<br />

r<br />

such that a + b + c = constant and abc = constant, then the sum a −1<br />

r + b −1<br />

r + c −1<br />

r is maximal<br />

when 0 < a ≤ b = c.<br />

So, it suffices to prove the inequality for 0 < a ≤ b = c; that is, to prove the homogeneous<br />

inequality in x, y, z for 0 < x ≤ y = z. Without loss of generality, we may leave aside the<br />

constraint xy + yz + zx = 3, and consider y = z = 1 and 0 < x ≤ 1. The inequality reduces to<br />

x r � � r<br />

2x + 1 2<br />

+ 2 ≥ 3<br />

.<br />

3<br />

Denoting<br />

f(x) = ln xr + 2<br />

3<br />

− r<br />

2<br />

2x + 1<br />

ln ,<br />

3<br />

J. Inequal. Pure and Appl. Math., 8(1) (2007), Art. 15, 21 pp. http://jipam.vu.edu.au/


18 VASILE CÎRTOAJE<br />

we have to show that f(x) ≥ 0 for 0 < x ≤ 1. The derivative<br />

f ′ (x) = rxr−1<br />

xr r<br />

−<br />

+ 2 2x + 1 = r(x − 2x1−r + 1)<br />

x1−r (xr + 2)(2x + 1)<br />

has the same sign as g(x) = x − 2x1−r + 1. Since g ′ (x) = 1 − 2(1−r)<br />

xr , we see that g ′ (x) < 0<br />

for x ∈ (0, x1), and g ′ (x) > 0 for x ∈ (x1, 1], where x1 = (2 − 2r) 1/r ≈ 0.416. The function<br />

g(x) is strictly decreasing on [0, x1], and strictly increasing on [x1, 1]. Since g(0) = 1 and<br />

g(1) = 0, there exists x2 ∈ (0, 1) such that g(x2) = 0, g(x) > 0 for x ∈ [0, x2) and g(x) < 0<br />

for x ∈ (x2, 1). Consequently, the function f(x) is strictly increasing on [0, x2] and strictly<br />

decreasing on [x2, 1]. Since f(0) = f(1) = 0, we have f(x) ≥ 0 for 0 < x ≤ 1, establishing<br />

the desired result.<br />

ln 9−ln 4<br />

Equality occurs for x = y = z = 1. Additionally, for p = and x ≤ y ≤ z, equality<br />

ln 3<br />

holds again for x = 0 and y = z = √ 3. �<br />

Proposition 3.15 ([7]). Let x, y, z be non-negative real numbers such that x + y + z = 3, and<br />

let p ≥ ≈ 0.29. Then,<br />

ln 9−ln 8<br />

ln 3−ln 2<br />

x p + y p + z p ≥ xy + yz + zx.<br />

Proof. For p ≥ 1, by Jensen’s inequality we have<br />

x p + y p + z p �<br />

x + y + z<br />

≥ 3<br />

3<br />

� p<br />

= 3 = 1<br />

3 (x + y + z)2 ≥ xy + yz + zx.<br />

ln 9−ln 8<br />

Assume now p < 1. Let r = and x ≤ y ≤ z. The inequality is equivalent to the<br />

ln 3−ln 2<br />

homogeneous inequality<br />

� �2−p x + y + z<br />

2(x p + y p + z p )<br />

3<br />

+ x 2 + y 2 + z 2 ≥ (x + y + z) 2 .<br />

By Corollary 1.9 (case 0 < p < 1 and q = 2), if x ≤ y ≤ z such that x + y + z = constant<br />

and x p + y p + z p = constant, then the sum x 2 + y 2 + z 2 is minimal when either x = 0 or<br />

0 < x ≤ y = z.<br />

Case x = 0. Returning to our original inequality, we have to show that y p + z p ≥ yz for<br />

y + z = 3. Indeed, by the AM-GM inequality, we get<br />

y p + z p − yz ≥ 2(yz) p<br />

2 − yz<br />

= (yz) p<br />

2 [2 − (yz) 2−p<br />

2 ]<br />

≥ (yz) p<br />

2<br />

= (yz) p<br />

2<br />

≥ (yz) p<br />

2<br />

� � � �<br />

2−p<br />

y + z<br />

2 −<br />

2<br />

� � � �<br />

2−p<br />

3<br />

2 −<br />

2<br />

� � � �<br />

2−r<br />

3<br />

2 − = 0.<br />

2<br />

J. Inequal. Pure and Appl. Math., 8(1) (2007), Art. 15, 21 pp. http://jipam.vu.edu.au/


EQUAL VARIABLE METHOD 19<br />

Case 0 < x ≤ y = z. In the homogeneous inequality, we may leave aside the constraint<br />

x + y + z = 3, and consider y = z = 1 and 0 < x ≤ 1. Thus, the inequality reduces to<br />

(x p � �2−p x + 2<br />

+ 2)<br />

≥ 2x + 1.<br />

3<br />

To prove this inequality, we consider the function<br />

f(x) = ln(x p x + 2<br />

+ 2) + (2 − p) ln − ln(2x + 1).<br />

3<br />

We have to show that f(x) ≥ 0 for 0 < x ≤ 1 and r ≤ p < 1. We have<br />

where<br />

and<br />

f ′ (x) = pxp−1<br />

x p + 2<br />

2 − p 2<br />

+ −<br />

x + 2 2x + 1 =<br />

2g(x)<br />

x 1−p (x p + 2)(2x + 1) ,<br />

g(x) = x 2 + (2p − 1)x + p + 2(1 − p)x 2−p − (p + 2)x 1−p ,<br />

g ′ (x) = 2x + 2p − 1 + 2(1 − p)(2 − p)x 1−p − (p + 2)(1 − p)x −p ,<br />

g ′′ (x) = 2 + 2(1 − p) 2 (2 − p)x −p + p(p + 2)(1 − p)x −p−1 .<br />

Since g ′′ (x) > 0, the first derivative g ′ (x) is strictly increasing on (0, 1]. Taking into account<br />

that g ′ (0+) = −∞ and g ′ (1) = 3(1 − p) + 3p 2 > 0, there is x1 ∈ (0, 1) such that g ′ (x1) = 0,<br />

g ′ (x) < 0 for x ∈ (0, x1)and g ′ (x) > 0 for x ∈ (x1, 1]. Therefore, the function g(x) is strictly<br />

decreasing on [0, x1] and strictly increasing on [x1, 1]. Since g(0) = p > 0 and g(1) = 0, there<br />

is x2 ∈ (0, x1) such that g(x2) = 0, g(x) > 0 for x ∈ [0, x2) and g(x) < 0 for x ∈ (x2, 1]. We<br />

have also f ′ (x2) = 0, f ′ (x) > 0 for x ∈ (0, x2) and f ′ (x) < 0 for x ∈ (x2, 1]. According to this<br />

result, the function f(x) is strictly increasing on [0, x2] and strictly decreasing on [x2, 1]. Since<br />

f(0) = ln 2 + (2 − p) ln 2<br />

2<br />

≥ ln 2 + (2 − r) ln = 0<br />

3 3<br />

and f(1) = 0, we get f(x) ≥ min{f(0), f(1)} = 0.<br />

ln 9−ln 8<br />

Equality occurs for x = y = z = 1. Additionally, for p = and x ≤ y ≤ z, equality<br />

ln 3−ln 2<br />

. �<br />

holds again when x = 0 and y = z = 3<br />

2<br />

Proposition 3.16 ([8]). If x1, x2, . . . , xn (n ≥ 4) are non-negative numbers such that x1 + x2 +<br />

· · · + xn = n, then<br />

1<br />

1<br />

1<br />

+<br />

+ · · · +<br />

≤ 1.<br />

n + 1 − x2x3 · · · xn n + 1 − x3x4 · · · x1 n + 1 − x1x2 · · · xn−1<br />

Proof. Let x1 ≤ x2 ≤ · · · ≤ xn and en−1 = � 1 + 1<br />

�n−1. By the AM-GM inequality, we have<br />

n−1<br />

� �n−1 � �n−1 x2 + · · · + xn x1 + x2 + · · · + xn<br />

x2 · · · xn ≤<br />

≤<br />

= en−1.<br />

n − 1<br />

n − 1<br />

Hence<br />

n + 1 − x2x3 · · · xn ≥ n + 1 − en−1 > 0,<br />

and all denominators of the inequality are positive.<br />

Case x1 = 0. It is easy to show that the inequality holds.<br />

Case x1 > 0. Suppose that x1x2 · · · xn = (n + 1)r = constant, r > 0. The inequality becomes<br />

x1 x2<br />

xn<br />

+ + · · · +<br />

x1 − r x2 − r xn − r<br />

≤ n + 1,<br />

J. Inequal. Pure and Appl. Math., 8(1) (2007), Art. 15, 21 pp. http://jipam.vu.edu.au/


20 VASILE CÎRTOAJE<br />

or<br />

1 1<br />

1 1<br />

+ + · · · + ≤<br />

x1 − r x2 − r xn − r r .<br />

By the AM-GM inequality, we have<br />

�<br />

x1 + x2 + · · · + xn<br />

(n + 1)r = x1x2 · · · xn ≤<br />

n<br />

hence r ≤ 1<br />

n+1 . From xn < x1 + x2 + · · · + xn = n < n + 1 ≤ 1<br />

we have r < xi < 1<br />

r<br />

for all numbers xi.<br />

� n<br />

= 1,<br />

r , we get xn < 1<br />

r<br />

. Therefore,<br />

We will apply now Corollary 1.7 to the function f(u) = −1<br />

u−r , u > r. We have f ′ (u) = 1<br />

(u−r) 2<br />

and<br />

g(x) = f ′<br />

� �<br />

1 x<br />

=<br />

x<br />

2<br />

(1 − rx) 2 , g′′ 4rx + 2<br />

(x) = .<br />

(1 − rx) 4<br />

Since g ′′ (x) > 0, g(x) is strictly convex on � r, 1<br />

�<br />

. According to Corollary 1.7, if 0 ≤ x1 ≤<br />

r<br />

x2 ≤ · · · ≤ xn such that for x1 + x2 + · · · + xn = constant and x1x2 · · · xn = constant, then the<br />

sum f(x1) + f(x2) + · · · + f(xn) is minimal when x1 ≤ x2 = x3 = · · · = xn. Thus, to prove<br />

the original inequality, it suffices to consider the case x1 = x and x2 = x3 = · · · = xn = y,<br />

where 0 < x ≤ 1 ≤ y and x + (n − 1)y = n. We leave ending the proof to the reader. �<br />

Remark 3.17. The inequality is a particular case of the following more general statement:<br />

Let n ≥ 3, en−1 = � 1 + 1<br />

�n−1, kn = n−1<br />

(n−1)en−1 and let a1, a2, . . . , an be non-negative<br />

n−en−1<br />

numbers such that a1 + a2 + · · · + an = n.<br />

(a) If k ≥ kn, then<br />

1<br />

1<br />

1<br />

+<br />

+ · · · +<br />

≤<br />

k − a2a2 · · · an k − a3a4 · · · a1 k − a1a2 · · · an−1<br />

n<br />

k − 1 ;<br />

(b) If en−1 < k < kn, then<br />

1<br />

1<br />

1 n − 1<br />

+<br />

+ · · · +<br />

≤<br />

k − a2a3 · · · an k − a3a4 · · · a1 k − a1a2 · · · an−1 k +<br />

1<br />

.<br />

k − en−1<br />

Finally, we mention that many other applications of the EV-Method are given in the book [2].<br />

REFERENCES<br />

[1] V. CÎRTOAJE, A generalization of Jensen’s inequality, Gazeta Matematică A, 2 (2005), 124–138.<br />

[2] V. CÎRTOAJE, Algebraic Inequalities - Old and New Methods, GIL Publishing House, Romania,<br />

2006.<br />

[3] V. CÎRTOAJE, Two generalizations of Popoviciu’s Inequality, Crux Math., 31(5) (2005), 313–318.<br />

[4] V. CÎRTOAJE, Solution to problem 2724 by Walther Janous, Crux Math., 30(1) (2004), 44–46.<br />

[5] V. CÎRTOAJE, Problem 10528, A.M.M, 103(5) (1996), 428.<br />

[6] V. CÎRTOAJE, Mathlinks site, http://www.mathlinks.ro/Forum/viewtopic.php?t=<br />

123604, Dec 10, 2006.<br />

[7] V. CÎRTOAJE, Mathlinks site, http://www.mathlinks.ro/Forum/viewtopic.php?t=<br />

56732, Oct 19, 2005.<br />

[8] V. CÎRTOAJE, Mathlinks site, http://www.mathlinks.ro/Forum/viewtopic.php?t=<br />

18627, Jan 29, 2005.<br />

[9] V. CÎRTOAJE, Mathlinks site, http://www.mathlinks.ro/Forum/viewtopic.php?t=<br />

14906, Aug 04, 2004.<br />

J. Inequal. Pure and Appl. Math., 8(1) (2007), Art. 15, 21 pp. http://jipam.vu.edu.au/


EQUAL VARIABLE METHOD 21<br />

[10] V. CÎRTOAJE, Mathlinks site, http://www.mathlinks.ro/Forum/viewtopic.php?<br />

t=19076, Nov 2, 2004.<br />

[11] V. CÎRTOAJE, Mathlinks site, http://www.mathlinks.ro/Forum/viewtopic.php?<br />

t=21613, Feb 15, 2005.<br />

[12] W. JANOUS AND V. CÎRTOAJE, Two solutions to problem 2664, Crux Math., 29(5) (2003), 321–<br />

323.<br />

[13] D.S. MITRINOVIC, J.E. PECARIC AND A.M. FINK, Classical and New Inequalities in Analysis,<br />

Kluwer, 1993.<br />

[14] J. SURANYI, Miklos Schweitzer Competition-Hungary. http://www.mathlinks.ro/<br />

Forum/viewtopic.php?t=14008, Jul 11, 2004.<br />

[15] http://www.mathlinks.ro/Forum/viewtopic.php?t=36346, May 10, 2005.<br />

[16] http://www.mathlinks.ro/Forum/viewtopic.php?p=360537, Nov 2, 2005.<br />

J. Inequal. Pure and Appl. Math., 8(1) (2007), Art. 15, 21 pp. http://jipam.vu.edu.au/

Hooray! Your file is uploaded and ready to be published.

Saved successfully!

Ooh no, something went wrong!